• 締切済み

ミックスド行列の存在がどうしても示せません

宜しくお願い致します。 ミックスド行列というものの存在が存在できません。 ミックスド行列の定義の前に先ず,ミックスド行列式を定義します。 [定義] S_n^+を正値対称行列全体の集合とします。そして A_1,A_2,…,A_n∈S_n^+, x_1,x_2,…,x_nを実変数とします。 この時,行列式 |x_1A_1+x_2A_2+…+x_nA_n|を展開した同次多項式の係数をD({A_1,A_2,…,A_n})という風にあらわし,ミックスド行列式と呼ぶ事にします。 つまり,ここでDは2^{S_n^+×{0,1,…,n}}からRへの写像となっています。 [具体例] 2×2行列にて: A,B∈S_2^+を A= a_1 a_2 =(a_1,a_2)^T B= b_1 b_2 =(b_1,b_2)^T (ここで,a_1,a_2,b_1,b_2は行ベクトルを表してます。テキスト表記の為,転置で表記することにします) とすると |xA+yB|=|(a_1,a_2)^T|x^2+(|(a_1,b_2)^T|+|(b_1,a_2)^T|)xy+|(b_1,b_2)^T|y^2 なので D({A,A})=|(a_1,a_2)^T|, D({A,B})=|(a_1,b_2)^T|+|(b_1,a_2)^T|, D({B,B})=|(b_1,b_2)^T| と求まります。 因みに, 下記の性質があることは定義から分かります。 [性質] D({A,B})=|A|, D({B,B})=|B|, Dは線形的,つまり, D({A+A',B})=D({A,B})+D({A',B}) ここでA'∈S_n^+, D({A,cB})=cD({A,B}) ここでcは定数, が成り立つ。 さらに,このDを陽関数表示すると, D({A_1,…,A_1,A_2,…,A_2,…,…,A_r,…A_r})というミックスド行列式は (ここで, A_1,A_2,…,A_rはそれぞれm_1,m_2,…,m_r個並んでるものとします。m_1+m_2+…+m_r=n) A_1からm_1個の行ベクトル,A_2からm_2個の行ベクトル,…,A_rからm_r個の行ベクトルをとって来てできる すべての行列式(このような行列式はn!/(m_1!m_2!…m_r!)個ある)の総和をn!/(m_1!m_2!…m_r!))で割ったものになります。 [具体例], 例1: A,B∈S_3^+でA=(a_1,a_2,a_3)^T, B=(b_1,b_2,b_3)^Tの時, D({A,A,B})=1/(3!/(2!1!)) (|(a_1,a_2,b_3)^T|+|(a_1,b_2,a_3)^T|+|(b_1,a_2,a_3)^T|) (この場合,m_1=2,m_2=1となります) D({B,B,B})=1/(3!/3!) |(b_1,b_2,b_3)^T|=|B|. (この場合,m_1=3となります) 例2: A,B,C∈S_3^+でA=(a_1,a_2,a_3)^T, B=(b_1,b_2,b_3)^T, B=(c_1,c_2,c_3)^Tの時, D({A,B,C})=1/(3!/(1!1!1!)) (|(a_1,b_2,c_3)^T|+|(a_1,c_2,b_3)^T|+|(b_1,a_2,c_3)^T|+|(b_1,c_2,a_3)^T|+|(c_1,a_2,b_3)^T|+|(c_1,b_2,a_3)^T|) (この場合,m_1=m_2=m_3=1となります) という具合になります。 次に, ミックスド行列の定義をします。 [定義] A_1,A_2,…,A_{n-1}∈S_n^+の時, ∃M∈S_n^+; for∀Q∈S_n^+, D({A_1,A_2,…,A_{n-1},Q})=D({M,M,…,M,Q}). というMをミックスド行列と呼ぶ. (ここで,M,M,…,Mは左辺と同じくn-1個並んでいます) このミックスド行列Mの存在がどうしても示せません。 どのようにすれば示せますでしょうか?

みんなの回答

  • stomachman
  • ベストアンサー率57% (1014/1775)
回答No.6

申し訳ないが、まだ出来てません。けれど、一番最初に回答した、主軸変換を使ったらどうか、という話をもう少し進めてみました。   D(A_1, A_2, …, A_n)   = Σ{j=0,1,…, n-1} ((-1)^j)Σ{s = A_1, A_2, …, A_nのうち(n-j)個の和}|s| です。さて、Qを正規直交行列U(U^T = U^(-1), |U|=1)を使って   Λ = U Q (U^T) と主軸変換して、対角行列Λを得たしたとします。このUを使って   B_k = U A (U^T) を作れば、   D(A_1, A_2, …, A_(n-1), Q) = D(B_1, B_2, …, B_(n-1), Λ) であることは明らか。  そこで、Qを任意の正値対称行列から取るのではなく、対角行列に限定して考えてみます。   S(A_1, A_2, …, A_n) = { C | p∈P_n ∧ C[i, j] = A_p[j][i, j] (i,j∈{1,2,…,n} } とすると   D(A_1, A_2, …, A_n) = Σ{C∈S(A_1, A_2, …, A_n)} |C| また   Rj(A) = Aからj行j列を除いた(n-1)×(n-1)行列   Λ = 対角行列 Λ[i,i] =λ[i] とすると   D(A_1, A_2, …, A_(n-1), Λ)    = Σ{j=1,…,n} λ[j] Σ{C∈S(Rj(A_1),Rj(A_2),…,Rj(A_(n-1)))} |C|   D(M, M, …, M, Λ)    = (n-1)! Σ{j=1,…,n} λ[j] |Rj(M)| である。そこで、   ∀(A_1, A_2, …, A_n) ∃M ∀Λ (D(A_1, A_2, …, A_(n-1), Λ)=D(M, M, …, M, Λ)) となるには   (n-1)! |Rj(M)| = Σ{C∈S(Rj(A_1),Rj(A_2),…,Rj(A_(n-1)))} |C| (j=1,2,…,n) でなくてはならない。n本の式に対して自由度(Mの成分)はn(n+1)/2個あるのだから、この方程式の解というだけならいくらでもあるに違いない、と分かります。  具体的にn=3の場合でやってみると、対角行列Λと対称行列Xについて   |X+Λ| - |X| - |Λ|   = λ[1](x[1,1]x[2,2]-x[1,2]^2) + λ[2]λ[3]x[1,1]   + λ[2](x[1,1]x[3,3]-x[1,3]^2) + λ[1]λ[3]x[2,2]   + λ[3](x[3,3]x[2,2]-x[2,3]^2) + λ[1]λ[2]x[3,3] だから、対称行列A, Bでは   D(A, B, Λ)   = |(A+B)+Λ| - |(A+B)| - |Λ| - (|A+Λ| - |A| - |Λ|) - (|B+Λ| - |B| - |Λ|)   = λ[1]c[1] + λ[2]c[2] + λ[3]c[3] ただし   c[1] = a[2,2]b[3,3]+a[3,3]b[2,2]-2a[2,3]b[2,3]   c[2] = a[1,1]b[3,3]+a[3,3]b[1,1]-2a[1,3]b[1,3]   c[3] = a[1,1]b[2,2]+a[2,2]b[1,1]-2a[1,2]b[1,2] であり、   D(M, M, Λ)   = 2 λ[1](m[2,2]m[3,3] - m[2,3]^2)   + 2 λ[2](m[1,1]m[3,3] - m[1,3]^2)   + 2 λ[3](m[1,1]m[2,2] - m[1,2]^2) である。  なので、方程式は   m[2,2]m[3,3] = m[2,3]^2 + c[1]/2   m[1,1]m[3,3] = m[1,3]^2 + c[2]/2   m[1,1]m[2,2] = m[1,2]^2 + c[3]/2 となります。ここで   W = m[1,1]m[2,2]m[3,3] とおけば   W/m[1,1] = m[2,3]^2 + c[1]/2   W/m[2,2] = m[1,3]^2 + c[2]/2   W/m[3,3] = m[1,2]^2 + c[3]/2 と書けて、解となる実行列Mが存在するのは明らか。  そこで、   Λ = U Q (U^T) に話を戻すと、残された問題は、「A, Bが正値行列であるとき、上記の方程式に(U^T) M U の全ての成分が正になる解Mがある」ということを証明する事。  たとえばUが単位行列の場合には、m[2,3] = if c[1]<0 then √(1-c[1]/2) else 1(以下同様)とすれば十分。だから、(U^T) M U の全ての成分を正に出来そうだという推測は付く。で、(ある対角行列を正値行列に主軸変換するような正規直交変換なら)どんなUでも大丈夫かどうか。  という方向でイケるのか? もっとスマートなアプローチがありそうな気もしますが。

BBeckyy666
質問者

お礼

すっかり遅くなりまして大変申し訳ありません。 拝読させていただいてます。 「重み」と「内挿」の意味が分からず頓挫してました。 ttp://golem.ph.utexas.edu/category/2011/08/mixed_volume.html から A_1,…,A_nを矩形Π_{k=1..n}[0,a_{i_k}] (但し,k=1,2,…,n)とすると, mixed volume V(A_1,…,A_n)は V(A_1,…,A_n)=1/n!Σ_{σ∈S_n}a_{1,σ(1)}…a_{n,σ(n)}という風に,n!個の矩形の組みあわせの和の平均(和をn!で割るので)で定義されるのですね。 これがどうして興味の対象になるのかピンと来ませんが。 > たとえば 2次元ユークリッド空間で、相異なる頂点a, b, cを持つconvexを考えますと、その内点pは >  p = w[1]a + w[2]b + w[3]c : >つまり、(1,0,0), (0,1,0), (0,0,1)を頂点とするconvexの内点から、a,b,cを頂点とするconvexの内点への写像がAです。 これは納得です。 > 同じ2次元ユークリッド空間で、2個の頂点d,eを持つconvexを考えるなら、それぞれを >  p = x[1]d + x[1]e >  ただし : >  y[1]=w[1], y[2]=w[2], y[3]=w[3], y[4]=x[1], y[5]=x[2] これは, A:= a_1,b_1,c_1 a_2,b_2,c_2 B:= d_1,e_1 d_2,e_2 E:= a_1,b_1,c_1,d_1,e_1 a_2,b_2,c_2,d_2,e_2 と置き, F:={(p_1,p_2)^T∈R^2;(p_1,p_2)^T=A(w_1,w_2,w_3)^T,w_1+w_2+w_3=1,0≦w_1,w_2,w_3≦1} G:={(p_1,p_2)^T∈R^2;(p_1,p_2)^T=B(x_1,x_2)^T,x_1+x_2=1,0≦x_1,x_2≦1} H:={(p_1,p_2)^T∈R^2;(p_1,p_2)^T=E(y_1,y_2,y_3,y_4,y_5)^T,y_1+y_2+y_3+y_4+y_5=1,0≦y_1,y_2,y_3,y_4,y_5≦1} とすると, > とすると、重みwもxもそれぞれの「ただし…」を自動的に満たす y_1:=w_1,y_2:=w_2,y_3:=w_3,y_4:=x_1,y_5:=x_2とすれば w_1+w_2+w_3=1∨x_1+x_2=1ならy_1+y_2+y_3+y_4+y_5=1を満たすので, F∪G⊂Hが言えることは理解しました。 > だから逆に、a,b,c,d,eを頂点とするconvexは、単に重みベクトルyの中の幾つかの > 成分だけを選んでconvexを作ることによって、「a,b,cを頂点とするconvexと、d, eを頂点 > とするconvexとの補間」として表せたことになります convexの補間(?). すいません。ここが良く分かりません。 "a,b,c,d,eを頂点とするconvex"は仮に(凹みの無い)五角形の周及び内部だとすると, "a,b,cを頂点とするconvex"は三角形の周及び内部で "d, eを頂点"は線分ですよね。 ここで「補間」の意味が分かりません。 > D(A_1, A_2, …, A_n) >  = Σ{j=0,1,…, n-1} ((-1)^j)Σ{s = A_1, A_2, …, A_nのうち(n-j)個の和}|s| これは試しにn=3の時,D(A,A,B)=1/(3!/(2!1!))|2A+B|-2|A+B|-|2A|+2|A|+|B| を計算してみましたが, どうしても 1/3(|(a_1,a_2,b_3)^T|+|(a_1,b_2,a_3)^T|+|(b_1,a_2,a_3)^T|) とならないのですが。。。勘違いしてますでしょうか? あと,分からない記号があります。 P_nとA_p[j][i,j]とは何を表すのでしょうか?

  • stomachman
  • ベストアンサー率57% (1014/1775)
回答No.5

 回答になりませんけど。  convexを混ぜ合わせる、という概念が面白いと思ったので、ちょっと考えました。  たとえば 2次元ユークリッド空間で、相異なる頂点a, b, cを持つconvexを考えますと、その内点pは   p = w[1]a + w[2]b + w[3]c   ただし   w[1]+w[2]+w[3]=1 ∧ 0≦w[1]≦1∧ 0≦w[2]≦1∧ 0≦w[3]≦1 と表せます。そこで、重みwを3次元ベクトルだと思えば、wは3次元ユークリッド空間中にある(1,0,0)^T, (0,1,0)^T, (0,0,1)^Tを頂点とするconvexの内点である。そして、a, b, cを並べた行列Aを考えれば   p = A w つまり、(1,0,0), (0,1,0), (0,0,1)を頂点とするconvexの内点から、a,b,cを頂点とするconvexの内点への写像がAです。  同じ2次元ユークリッド空間で、2個の頂点d,eを持つconvexを考えるなら、それぞれを   p = x[1]d + x[1]e   ただし   x[1]+x[2]=1 ∧ 0≦x[1]≦1∧ 0≦x[2]≦1 と表せる。そして、相異なる点a,b,c,d,eを頂点とするconvexは   p = y[1]a + y[2]b + y[3]c+y[4]d + y[5]e   ただし   y[1]+y[2]+y[3]+y[4]+y[5]=1 ∧   0≦y[1]≦1∧ 0≦y[2]≦1∧ 0≦y[3]≦1 ∧ 0≦y[4]≦1∧ 0≦y[5]≦1 と表せる。ここで、重みyを   y[1]=w[1], y[2]=w[2], y[3]=w[3], y[4]=x[1], y[5]=x[2] とすると、重みwもxもそれぞれの「ただし…」を自動的に満たす。だから逆に、a,b,c,d,eを頂点とするconvexは、単に重みベクトルyの中の幾つかの成分だけを選んでconvexを作ることによって、「a,b,cを頂点とするconvexと、d, eを頂点とするconvexとの補間」として表せたことになります。そして、重みベクトルが作るconvexは単位行列の縦ベクトルを頂点とするconvexになっていて、頂点ベクトルa,b,d,eを並べて作った行列をAとすれば、   p = A y は「単位行列の縦ベクトルを頂点とするconvexから、Aの縦ベクトルを頂点とするconvexへの、写像」になっている。  このconvexを、頂点が重複している複数のconvex、たとえば「a,b,cを頂点とするconvexと、a,b,d,eを頂点とするconvexの補間」と見ることもできます。すなわち、後者のconvexの内点を   p = z[1]a + z[2]b + z[3]d + z[4]e   ただし   z[1]+z[2]+z[3]+z[4]=1 ∧ 0≦z[1]≦1∧ 0≦z[2]≦1∧ 0≦z[3]≦1∧ 0≦z[4]≦1 として、   y[1]=w[1]=z[1], y[2]=w[2]=z[2], y[3]=w[3], y[4]=z[4], y[5]=z[5] とすれば良い。つまり、共通の頂点の重みは、共通の成分で表現する、ということです。  a,b,cを頂点とするconvexの内点qについて、q,d,eを頂点とするconvexとa,b,cを頂点とするconvexを補間したらどうなるか。これはもちろん、「a,b,cを頂点とするconvexとq, d,eを頂点とするconvexとの補間」である。けれどもこれは「a,b,cを頂点とするconvexとd,eを頂点とするconvexとの補間」と同じである。つまり、頂点qがa,b,cの内点になっていればqは取り除ける。(アナロジーですけど、線形部分空間同士の直積を作るのに似てる感じです。というのは、線形従属の代わりに内挿になってるかどうかが問われる。)  以上の話は、2次元空間のconvexに限らず、何次元でも同じです。  一般に、あるconvexについて、重みベクトルの成分を幾つか選んでconvexを作る。同様にそのようなconvexを幾つか作る。すると、重みベクトルのどの成分も少なくともどれかひとつのconvexに含まれていさえすれば、元のconvexを「幾つかのconvexの補間」として表せたことになる。  また、一般に、k個の頂点を持つconvexはどれでも、その内点の重みはk次元空間中のk-1次元超平面上にある頂点(1,0,…,0), …, (0,…,0,1)を持つconvexに対応している。  という訳で、convexを扱うには、内点を表す重みベクトルの空間を考えれば話が単純化できそうだなあ、と思います。  あ、いや、それだけの話なんですけどね。

BBeckyy666
質問者

お礼

また遅くなりまして申し訳ありません。 >> D(A_1, A_2, …, A_n) >>  = Σ{j=0,1,…, n-1} ((-1)^j)Σ{s = A_1, A_2, …, A_nのうち(n-j)個の和}|s| > これは試しにn=3の時,D(A,A,B)=1/(3!/(2!1!))|2A+B|-2|A+B|-|2A|+2|A|+|B| > を計算してみましたが, > どうしても > 1/3(|(a_1,a_2,b_3)^T|+|(a_1,b_2,a_3)^T|+|(b_1,a_2,a_3)^T|) > とならないのですが。。。勘違いしてますでしょうか? これは大変失礼いたしました。 D(A_1, A_2, …, A_n)   = 1/(n!/(1!1!…1!))Σ{j=0,1,…, n-1} ((-1)^j)Σ{s = A_1, A_2, …, A_nのうち(n-j)個の和}|s| と平均をとる1/(n!/(1!1!…1!)を付け加えて実際に数値計算で上手くいきました。\(^o^)/ > さて、Qを正規直交行列U(U^T = U^(-1), |U|=1)を使って ; > に話を戻すと、残された問題は、「A, Bが正値行列であるとき、上記の方程式に > (U^T) M U の全ての成分が正になる解Mがある」ということを証明する事。 に関しては今暫くお時間をください。 ところで,正値対称行列についてはミックスド行列が存在する事は知られているようなのですが, 今,訳あって, 正値対称行列上でのミックスド行列の存在が保証されてる状況下で 正値エルミート行列についてはミックスド行列が存在することを示したいのですが, 何かいい方法はありませんでしょうか? 双方とも,対角行列の対角成分(固有値)が正の実数なので 正値対称行列上でのミックスド行列の存在性から 正値エルミート行列上でのミックスド行列の存在性は簡単に言えるのではと思ったのですが頓挫してしまいました。

  • stomachman
  • ベストアンサー率57% (1014/1775)
回答No.4

ANo.3へのコメントについて。  話が一気に拡散したようで、どひー。さっぱり付いて行けんですが、ともあれ、Theorem5.1.6のmixed bodies というのは、複数のconvexの重み付き平均(ただし重みの合計は1、つまり内挿でしょう。コメント内の引用はミスプリがあるのでは?)で出来てるconvexのこと。定理がその存在を言っている「non-negative symmetric function V:(K^n)^n→R」というのが、「n×n正値対称行列の行列式」、つまり「ミックスド行列式」のことでしょうかね。  convexでは、頂点の内挿(つまり重み付き平均。ただし、重みは非負で、合計が1)によって表せる点の集合が内点の集合に他ならない。(もちろん、各表面に対応する一次不等式を連立したものでも表せますが。)n次元なら原点を含めてn+1個の頂点ベクトルを選ぶと、原点以外のn個を並べてn×n行列を作ったとき、その行列式が0でなければ自動的にconvexになっている。しかし、n+1個ではなく2^n個の頂点を持つconvexを考えているんですから、それがconvexになるためには、どの頂点も他の頂点たちの内挿では表せない、というのが必要十分条件。頂点ベクトルを並べた行列A_i,jはn行(2^n)列になる。重みベクトルも(2^n)次元のベクトルになるわけで、ある内点に対応する重みベクトルはもちろん一意的には決まらない。となると、なんだか一般化逆行列の話と関係しそうだなあ、という気がします。ならば、固有値で分解するのも、そんなに大はずれではないかも。

  • stomachman
  • ベストアンサー率57% (1014/1775)
回答No.3

ANo.2へのコメントについてです。 > A_1,A_2,…,A_nが同一の正則行列で対角化できれば本当にシンプルに出来ると思いますが もちろんそうは行きませんよね。えっとですね、たとえばn=3の場合、   (3!) D({A,B,Q}) = |A+B+Q|-|A+B|-|B+Q|-|Q+A|+|A|+|B|+|Q| から   (3!) D({M,M,Q}) = |2M+Q| -|2M| -2|M+Q| +2|M| +|Q|   = |2M+Q| -6|M| -2|M+Q| +|Q| である。ここで   M = (U^T)ΛU (ただし Uは正規直交行列、Λは対角行列) と分解して   R = UQ(U^T) とおけば   (3!) D({M,M,Q}) = |2Λ+R| -6|Λ| -2|Λ+R| +|R| となる。   Z^n = {R | ∃Q∃M∃Λ∃U(Q∈S_n^+ ∧ なんたらかんたら) } とでもすれば、勝手なA_1, A_2∈S_n^+ に対して存在するM: s.t.   M∈S_n^+ ∧ ∀Q(Q∈S_n^+ ⇒ D({M,M,Q}) = D({A_1,A_2,Q})) のQをR(∈Z^n)で、MをΛで置き換えられるんじゃ?という話です。が、いや、どうも何か見当違いのこと言ってるかもです。

BBeckyy666
質問者

お礼

遅くなりまして申し訳ありません。 悪あがきかもしれませんが、、 ミックスド行列式とミックスド体積の関係を述べてるサイトを見つけました。 golem.ph.utexas.edu/category/2011/08/mixed_volume.html 最後ら辺の 『I’m confused about which analogy to trust. I had been thinking of convex bodies in …』とかも詠んでみました。 ちなみに凸体の定義は次のとおりです。 凸集合とはその集合内の2点を結ぶ線分は必ずその集合に含まれる集合の事 です。 凸体(convex body)とは凸集合(2次元)の高次元版のようです。 開集合で任意の2点を結ぶ線分はその集合内に含まれるもの,若しくは空でないコンパクトなE^n内の凸集合の事。 さて, Vをn^2個の頂点を持つ凸体とする時, そのn個の頂点の縦座標(n項行ベクトル)を並べてできる行列A:=(a_ij)とを対応させてるようです。 n×n正値対称行列の集合とR^n内の凸体の集合との間に一体どのような同型写像がありそうでしょうか? ちなみに Convex Bodies: The Brunn?Minkowski Theory 著者: Rolf Schneider 1993年 にミックスド体についての記述がありました。 『By K^n we denote the set of all convex bodies in E^n and by K_0^n the subset of convex bodies with interior points.』 『Mixed bodies: For convex bodies K_1,…,K_{n-1}∈K_0^n, the mixed area measure S(K_1,…,K_{n-1},・) satisfies the assumptions of Minkowski's theorem: hence there is a convex body [K_1,…,K_{n-1}], unique up to a translation, for which S_{n-1}([K_1,…,K_{n1}],・)=S(K_1,…,K_{n-1},・). [K_1,…,K_{n1}] is called the mixed body of K_1,…,K_{n-1}.』 『h(K,u):=sup{<x,u>;x∈K} for u∈E^n, <,> is the scalar product』 『Theorem5.1.6(and Definition). There is a non-negative symmetric function V:(K^n)^n→R, the mixed volume, such that V_n(λ_1K_1+…+λ_mK_m)=Σ_{i_1,…,i_n=1}^mλ_{i_1}…λ_{i_n}V(K_{i_1},…,K_{i_n}) for arbitrary convex bodies K_1,…,K_m∈K^n and numbers λ_1,…,λ_m≧0. Further, there is a symmetirc map S from (K^n)^{n-1} into the space of finite Borel measure on S^{n-1}, the mixed area measure, such that S_{n-1}(λ_1K_1+…+λ_mK_m,・)=Σ_{i_1,…,i_n=1}^mλ_{i_1}…λ_{i_{n-1}}S(K_{i_1},…,K_{i_{n-1}},・) for K_1,…,K_m∈K^n and λ_1,…,λ_m≧0, where S(K_1,…,K_{n-1},・):=S(K_1,…,K_{n-1})(・). The equality V(K_1,…,K_n)=1/n∫_{S^{n-1}}h(K_1,u)dS(K_2,…,K_n,u) holds for K_1,…,K_n∈K^n』 さらに下記の改訂版(2014年)にも詳しく載ってます。 googleで「"mixed area measure" is said」と入力したら この書籍のデジタルバージョンが閲覧できます。 books.google.com/books?id=VfbXAAAAQBAJ&pg=PA330&dq=%22mixed+area+measure%22+is+said&hl=ja&sa=X&ei=elOhU-WgKPHfsAS-4YGgDA&ved=0CB0Q6AEwAA#v=onepage&q=%22mixed%20area%20measure%22%20is%20said&f=false p330

BBeckyy666
質問者

補足

大変有難うございます。 ミックスド行列式を一般的に陽関数表示すると A_1,A_1,…,A_1,A_2,…,A_2,……,A_r,…,A_rにてA_1がm_1個,A_2がm_2個,…,A_rがm_r個並んでいる時, (ここでm_1+m_2+…+m_r=n) (A_1,m_1),(A_2,m_2),…,……,(A_r,m_r)と表すことにすれば (D({(A_1,m_1),(A_2,m_2),…,……,(A_r,m_r}):=)D({A_1,A_1,…,A_1,A_2,…,A_2,……,A_r,…,A_r}) =1/(n!/(m_1!m_2!…m_r!))Σ_{v_1,…,v_r∈{0,1}^n,v_1+…+v_r=(1,1,…,1)^T∈R^n,∥v_1∥=m_1,…,∥v_r∥=m_r} |diag(a_{1_1},a_{1_2},…,a_{1_n}),diag(a_{2_1},…,a_{2_n}),…,diag(a_{r_1},…,a_{r_n})(v_1,v_2,…,v_r)^T| (但し, (a_{i_1},a_{i_2},…,a_{i_n})^T:=A_i, i=1,2,…,r, つまり,a__{i_j}と書いたらA_iの第j行ベクトルを表している) と書けます。ここで diag(a_{1_1},a_{1_2},…,a_{1_n}),diag(a_{2_1},…,a_{2_n}),…,diag(a_{r_1},…,a_{r_n})(v_1,v_2,…,v_r)^T =diag(a_{1_1},a_{1_2},…,a_{1_n})v_1+diag(a_{1_1},…,a_{1_n})v_2+…+diag(a_{r_1},a_{r_2},…,a_{r_n})v_r となります。 アダマード積の記号・を使って表記すると (D({(A_1,m_1),(A_2,m_2),…,……,(A_r,m_r}):=)D({A_1,A_1,…,A_1,A_2,…,A_2,……,A_r,…,A_r}) =1/(n!/(m_1!m_2!…m_r!))Σ_{v_1,…,v_r∈{0,1}^n,v_1+…+v_r=(1,1,…,1)^T∈R^n,∥v_1∥=m_1,…,∥v_r∥=m_r} |(a_{1_1},a_{1_2},…,a_{1_n})^T・v_1+(a_{2_1},…,a_{2_n})^T・v_2+…+(a_{r_1},a_{r_2},…,a_{r_n})^T・v_r| とも書けますね。 > (3!) D({A,B,Q}) = |A+B+Q|-|A+B|-|B+Q|-|Q+A|+|A|+|B|+|Q| A=I,B=2I, Q=3I (I:単位行列)の場合, |Ax+By+Qz|=|(x+2y+3z)I| =(x+2y+3z)^3|I|=(x+2y+3z)^3 =x^3+6x^2y+9x^2z+12xy^2+36xyz+27xz^2+8y^3+36y^2z+54yz^2+27z^3 より, D({A,B,Q}) = 1/3! 36(∵3!D({A,B,Q})はxyzの係数の事だから) =6 一方, (3!) D({A,B,Q}) = (3!) D({I,2I,3I}) =3! (|I+2I+3I|-|I+2I|-|2I+3I|-|3I+I|+|I|+|2I|+|3I|) =3! (|6I|-|3I|-|5I|-|4I|+|I|+|2I|+|3I|) =3! (6^3-3^3-5^3-4^3+1^3+2^3+3^3) =3! 36より, D({A,B,Q})=6 と確かに一致しますね。これには気づきませんでした。さすがです。 > である。ここで >   M = (U^T)ΛU (ただし Uは正規直交行列、Λは対角行列) : > のQをR(∈Z^n)で、MをΛで置き換えられるんじゃ?という話です。 > が、いや、どうも何か見当違いのこと言ってるかもです。 ありがたく拝見させていただいてます。 これは余談ですが ミックスド行列式,ミックスド行列は凸体でのミックスド体積,ミックスド凸体と連動していて, ミックスド体積,ミックスド凸体の考察が困難なケースが多いので,各ミックスド凸体に対応する正値対称行列を使って,間接的に考察するようです。 るようです。 なので 凸体の集合と正値対称行列の集合との間に同相関係があることが言えれば, "凸体V_1,V_1,…,V_1,V_2,…,V_2,……,V_r,…,V_rに対して,ミックスド凸体Vが在る" ⇔ "正値対称行列A_1,A_1,…,A_1,A_2,…,A_2,……,A_r,…,A_rに対して,ミックスド凸体Mが在る" が成り立つのではとも推測しましたが,同相(位相同形)であることの証明が分かりません。

  • stomachman
  • ベストアンサー率57% (1014/1775)
回答No.2

ANo.1へのコメントについてです。 > これを使えば 自然にA^αが定義できますね いや、そういうんじゃなくてですね、|UΛU^T|=|Λ|=(Λの対角成分の積) だから行列式がうんと簡単になるでしょ、という話なんです。

BBeckyy666
質問者

お礼

有難うございます。 > いや、そういうんじゃなくてですね、|UΛU^T|=|Λ|=(Λの対 > 角成分の積) だから行列式がうんと簡単になるでしょ、とい > う話なんです。 これは仰る通りですね。 A_1,A_2,…,A_nが同一の正則行列で対角化できれば本当にシンプルに出来ると思いますが,そのような保証はありませんよね。

  • stomachman
  • ベストアンサー率57% (1014/1775)
回答No.1

いや、まだ考えてないですけど。 もしかして、Aを   A = U Λ U^T (Λ:対角行列, U:正規直交行列) としたらもう少し話が整理できるんでは?

BBeckyy666
質問者

補足

失礼致しました。訂正箇所がありました。 |x_1A_1+x_2A_2+…+x_nA_n|を展開した同次多項式の係数をD({A_1,A_2,…,A_n}) ↓ |x_1A_1+x_2A_2+…+x_nA_n|を展開した同次多項式の係数を(n!/(m_1!m_2!…m_r!))D({A_1,A_2,…,A_n}) でした。これでDの陽関数表示と一致します。 つまり,D({A_1,A_2,…,A_n})は係数の平均値を取っている事になります。 (※余談ですが回路の並列繋ぎの計算に用いられるとからしいです) > もしかして、Aを >   A = U Λ U^T (Λ:対角行列, U:正規直交行列) > としたらもう少し話が整理できるんでは? 対称行列なので直交行列で A=UΛU^T ( = U diag(λ_1,λ_2,…,λ_n)U^T,λ_1,…,λ_nはAの固有値) とできますね。 これを使えば 自然にA^αが定義できますね(αは実数)。 具体的には A^α := U diag(λ_1^α,λ_2^α,…,λ_n^α)U^T と定義すればよい. A_k=U_k diag(λ_{k_1},λ_{k_2},…,λ_{k_n})U_k^T (ここでU_kは直交行列,k=1,2,…,n) とすると, |x_1A_1+x_2A_2+…+x_nA_n| =|x_1U_1Λ_1U_1^T+x_2U_2Λ_2U_2^T+…+x_nU_nΛ_nU_n^T| =|U_1(x_1Λ_1)U_1^T+U_2(x_2Λ_2)U_2^T+…+U_n(x_nΛ_n)U_n^T| といった感じでしょうか。 A_1,A_2,…,A_nが同じ正則行列で対角化される保証はありませんよね。 P.S. Aがエルミート行列の場合も対角成分が実数でユニタリ行列Uで A = U diag(λ_1,λ_2,…,λ_n)U^* と分解できるので,下記のような定義でもMが存在するのではと思っています。 D({A_1,A_2,…,A_{n-1},Q})=D({M,M,…,M,Q})の両辺を展開した時(相当複雑な方程式になりますが)に 複素数(代数的閉体)の範囲なら,Mについて解けるのではとも思いました。 [定義] A_1,A_2,…,A_{n-1}∈H_n^+の時, ∃M∈H_n^+; for∀Q∈H_n^+, D({A_1,A_2,…,A_{n-1},Q})=D({M,M,…,M,Q}). というMをミックスド行列と呼ぶ. ここで,H_n^+はエルミート正値行列全体の集合を表します。

関連するQ&A

  • 行列の問題です。

    行列の問題です。 A^tはAの転置行列 R^nの2つのベクトル x^t=(x_1,..,x_n) y^t=(y_1,..,y_n) に対して内積<x,y>を Σ_{i=1~n}x_iy_i で定義する。 Aをn×n実交代行列とする。 Bをすべての固有値が正となる実対称n×n行列とする。 (1)任意のベクトルx∈R^nに対して <Ax,x>=0を示せ。 (2)任意のベクトルx∈R^nに対して <Bx,x>≧0であり、 統合はx=0のときに限ることを示せ。 (3)A+Bは正則行列となることを示せ。 よろしくお願いします。

  • 表現行列

    Vを実数に係数を持つ2次以下の多項式全体が成すベクトル空間とする。すなわち、 V={a+bx+c*x^2|a、b、c∈R} である。tを0≦t なる定数とし、線形変換T :V→V を T(f(x))=f(1+tx)により定義する。 Vの基底1、x、x^2に関するTの表現行列を求めよ。 という問題があります。一般に、、、、 【線形写像f:R^n→R^mに対して、(m,n)型の行列Aがただひとつ定まり、 x'=f(x)=Axと表せる。(x∈R^n, x'∈R^m) この行列Aを、線形写像fの表現行列という。】 表現行列はこのように定義されていますから、この問題の場合 t^(T(1),T(x),T(x^2))= (1,0,0) (1,t,0) (1,2t,t^2) * t^(1,x,x^2) となるため、求める表現行列Aは (1,0,0) (1,t,0) (1,2t,t^2) となるかと思っていたのですが、解答には、これを転置した行列が書いてありました。 (1,1,1) (0,t,2t) (0,0,t^2) となっていました。 なぜこうなるのか理屈が分からないのですみませんが教えてください。

  • 行列の問題!

    すみません、、、どなたか以下の問題を解いて頂けませんか。 答えがないので、正直お手上げ状態です。 ヒントでも構いません、、、、宜しくお願いします。 ------------------------------------------------------------------ すべての実数からなる集合をRと表す。 行列を要素にもつ2つの集合M, Nを M = | | a b | a, b, c, d ∈ R |   .   | | c d |            | N = | | r -s | r, s ∈ R | ..  | | s r  .|        .| と定める。更に、Mの要素 A = | a b | に対し、 A' = | a c | とおく。  ....  | c d |          .| b d | (1) A, B ∈ N ならば、AB ∈ N であることを示せ。 (2) A, B ∈ N ならば、(AB)' = A'B' となることを示せ。   また、(CD)' ≠C'D' となる M の要素 C, D の組を一つ求めよ。 (3) N のすべての要素と交換可能な M の要素は、必ず N に属することを示せ。   ただし、行列 X と Y が交換可能であるとは、 XY = YX のこととする。 ------------------------------------------------------------------

  • x=Ux'という形を満たすn×n直交行列Uの存在?

    こんにちは。 R^n∋x:=(x_1,x_2,…,x_n)^T≠0をn次ベクトルとすると, x=Ux' (ただし,x':=(x'_1,x'_2,…,x'_{n-m},0,…,0)^T∈R^n, 1<m<n)という形を満たす n×n直交行列Uの存在を示したいのですがどうすればいいでしょうか? なお, 「^T」は転置を表します。

  • 行列式の答えに辿り着けません

    次の行列式を計算せよ。 |a b c d| |b a d c| |c d a b| |d c b a| …本の答えは (a+b+c+d)(a-b+c-d)(a+b-c-d)(a-b-c+d) となっています。 自分でやってみました。 サラスで a^4 + b^2・d^2 + c^4 + b^2・d^2 - d^4 - a^2・c^2 - b^4 - a^2・c^2 = a^4 - 2(a^2・c^2) + c^4 - {b^4 - 2(b^2・d^2) + d^4} p=a^2, q=b^2, r=c^2, s=d^2としますと = p^2 - 2pq + q^2 - (r^2 - 2rs + s^2) = (p-q)^2 - (r-s)^2 m=p-q, n=r-sとしますと = m^2 - n^2 = (m-n)(m+n) = {(p-q)-(r-s)}{(p-q)+(r-s)} = {(a^2 - b^2)-(c^2 - d^2)}{(a^2 - b^2)+(c^2 - d^2)} = {(a-b)(a+b)-(c-d)(c+d)}{(a-b)(a+b)+(c-d)(c+d)} …となりました。 まず、ここまでは合っていますでしょうか? 合っているならば、ここからどうやって本の答えまで辿り着けるのでしょうか? まさか全部掛け合わせるとか言いませんよね? ではお願いします。

  • 行列の問題です

    行列A=(2 3 1 2),P=(√3 -√3 1 1)に対して、B=(P^-1)APとおく。また、n=1,2,3,・・に対して、a_n,b_nを (a_n b_n)=A^n(2 0)で定める。(これは列で表しています) (1)P^-1とBを求めよ。 (2)a_n,b_nを求めろ。 (3)実数xを超えない最大の整数を[x]で表す。このとき[(2+√3)^n]=a_nー1を示せ。 また、c_n=(2+√3)^nー[(2+√3)^n]とするとき、lim(n→∞)c_nを求めよ。   行列はa,b,c,dの順で表しています 解説できる方おねがいします。

  • 次の表現行列は(実)ユニタリである事を示せ

    VをR上の有限次元内積空間とする. [問] Rを実数体とする。VをR上の有限次元内積空間とする。 B:={v_1,v_2,…,v_n}とB':={w_1,w_2,…,w_n}を夫々,Vの正規直交基底とする。 f:V→Vを線形写像とする時, 基底BとB'に関するfの表現行列をM_B_B'(f)で表す。 (1) id:V→Vを恒等写像とすると,M_B_B'(id)は実ユニタリ(直交行列(?))であることを示せ。 [ヒント:<w_i,w_i>=1,i≠jなら<w_i,w_j>=0.また表現w_i=Σa_ijv_j (a_ij∈R)] (2) f:V→Vをf(v_i)=w_i (i=1,2,…,n)とすると,M_B_B'(f)はユニタリであることを示せ。 と言う問題です。 これらはどのようにして求めればいいのでしょうか? (1)については 表現行列の定義から x=Σa_iv_i (a_1,a_2,…,a_n∈R)とするとM_B_B'(id)(x)=M_B_B'(x) (∵恒等写像の定義) =Σ[i=1..n]c_iw_i (但し,c_1,c_2,…,c_n∈R) と書け、 ユニタリの定義から内積が保存される事,つまり <M_B_B'(id)(x),M_B_B'(id)(y)>=<x,y>を示せばいいのだと思います。 y=Σb_iv_i (b_1,b_2,…,b_n∈R)として, M_B_B'(id)(y)=Σ[i=1..n]d_iw_i (但し, d_1,d_2,…,d_n∈R) とすると <M_B_B'(id)(x),M_B_B'(id)(y)>=<Σ[i=1..n]c_iw_i,Σ[i=1..n]d_iw_i> =Σ[i=1..n]<c_iw_i,d_iw_i> (∵直交の定義) =Σ[i=1..n]c_id_i (∵正規の定義) となり,<x,y>から遠ざかっております。 どのようにして証明すればいいのでしょうか? (2)についてはユニタリの定義はノルムを保存する事 <M_B_B'(f)(x),M_B_B'(f)(x)>=<x,x> を示す事だと思います。 M_B_B'(f)(x)=M_B_B'(f)(Σa_iv_i)=M_B_B'(f(Σa_iv_i)=Σ[i=1..n]a'_iw_i M_B_B'(f)(y)=M_B_B'(f)(Σb_iv_i)=M_B_B'(f(Σb_iv_i)=Σ[i=1..n]b'_iw_i となり,=<x,x>にたどり着けません。どうすればいいのでしょうか?

  • 行列式

    X=[AB]   [0D] というm+n次正方行列(Aはm次正方、Bはn次正方)に対し、 |X|=|A||D|が成り立つ理由をできるだけ容易なやり方で教えてください。

  • 逆行列の公式 証明お願いします

    逆行列の公式 証明お願いします n*n A Aは正則 n*m B m*n C m*m D (A+BC)^-1 = A^-1 - A^-1*B(I+CA^-1*B)^-1*CA^-1 A=Iなら  I - B(I+CB)^-1*C   = I - BC(I+BC)^-1 = I - (I+BC)^-1*BC 上記のものと S = D - CA^-1*Bとおき det(A)≠0 det(S)≠0のとき |A B|^-1 = |A^-1 + A^-1*BS^-1*CA^-1   -A^-1*BS^-1| |C D|     |    -S^-1*CA^-1              S^-1 | ||でくくったのは行列式ではなく 行列の行列です こうなるらしいのですが 証明など書かれておらず頭も悪いのでどうすればいいかもわかりません ヒントでもいいので教えていただけませんか?

  • 広義重積分と行列の数列

    2問お聞きしたいのですが 1問目は ε>0としD={(x,y)∈R^2|ε^2≦x^2+y^2≦1} lim(ε→0)∬_D{(x^2-y^2)/(x^4+y^4)}dxdyを求めよ という問題で,極座標へ変数変換したまではいいんですが, (cos^2θ-sin^2θ)/(cos^4θ+sin^4θ)の積分が出来なくて…。 2問目は(Aの右に付けてる数字とnは数列の番号です) A0=((1 0)^t (1 1)^t) (Aは行列,^tは転置)  A1=((a c)^t (b d)^t) (a,b,c,d∈R | 0<c<1 | ad-bc=1) An=((an cn)^t (bn dn)^t) A(n+1)=(An)(A0)(An^-1) M=1/(1-|c|)とし,|a|<Mと仮定する。 (1)andn-bncn=1が成り立つことを示せ。 (2)cnを求めよ。 (3)|an|<Mを証明せよ。  (1),(2)はできたんですが,(3)がどう手を付けて良いのかわかりません。 どなたか解説お願いします。